My issue with this stimulus is about the term consent versus consult. If ANY of the members had said "No" to the release of this report, would that still mean the chairperson consulted them?
http://7sage.com/lsat_explanations/lsat-49-section-2-question-12/
Just wondered if anyone could provide some insight into how they approached this question.
I fell for the trap presented in E, I took "interest rate hikes generally ...
Anybody know why the correct Answer choice B weakens the argument??I just can't seem to see how communal objects being passed down from one generation to the next weakens the argument.
I had this down to B or D during the timed exam, and I can't figure out what is technically wrong with B. In my mind, it's as close to a sufficient assumption as D is.
https://7sage.com/lsat_explanations/lsat-50-section-2-question-12/
Hey 7Sagers, I just did this question and did almost everything right, but ultimately chose (E). I understood there to be 2 gaps,the first between: ~being able to tell ulterior ...
https://7sage.com/lsat_explanations/lsat-73-section-2-question-21/
Hi, I was wondering whether anyone could explain why E is correct.
I was not sure about E because it says more than half of the students received a grade of B-or higher, but I ...
https://7sage.com/lsat_explanations/lsat-68-section-2-question-12/
I was not sure between B and D...
As for D, I thought the word "It fails to address..." Does the argument have to address it? I thought if so to make an argument people would ...
Hello,
I have a question about these two choices
1. the argument takes for granted that the higher sales of established products are due to effective advertising
2. the argument confuses a condition necessary for increasing product ...
Ok so I'm doing my blind review, and when I initially did the test I chose the correct answer even though I wasn't happy with it because it seemed more like a sufficient assumption to me. I don't have the ultimate package so I don't have the explanation ...
Hi guys I would appreciate it if someone could review my reasoning for this question and let me know if this is correct. When BRing I realized that I made a mistake and chose A rather than E which is correct.
I don't get the explanation for why A is right. All we know is that do produce a good meal you can't have bad food. That would seem to imply that you could have a good meal with mediocre food. So how can you then take the the next conditional relationship ...